LSAT and Law School Admissions Forum

Get expert LSAT preparation and law school admissions advice from PowerScore Test Preparation.

 Administrator
PowerScore Staff
  • PowerScore Staff
  • Posts: 8917
  • Joined: Feb 02, 2011
|
#26461
Complete Question Explanation

Question #1: Main Point. The correct answer choice is (B)

The economist believes that efforts to increase profitability do not always benefit businesses, because such efforts can cause employees to get dismissed. The main conclusion can be found in the second sentence, which begins with the counterpoint indicator “but” and is clearly supported by the last sentence. As with many Main Point questions, the main conclusion is not in the last sentence of the stimulus.

Like other Main Point questions, here the stimulus is more complex than usual. For instance, the first sentence introduces and defends a business practice, only to undermine it in the second sentence:
  • Premise: Attempts to increase productivity can have deleterious effects on employees.

    Conclusion: Not all efforts to increase productivity and beneficial to the business as a whole.
Answer choice (A): Even if this principle can be properly inferred from the information contained in the stimulus, it does not capture the main point of the argument. The economist’s conclusion focuses specifically on whether efforts to increase productivity, not general actions taken to secure the survival of a business, can be good for the business as a whole.

Answer choice (B): This is the correct answer choice, as it restates the conclusion in the second sentence of the stimulus. Note that the phrase “not all” is synonymous with the phrase “some are not” (or “some… fail to”).

Answer choice (C): Whether the employees of a business are also its owners is not a relevant consideration in this argument, let alone its main conclusion.

Answer choice (D): While it is true that every business strives to increase its productivity (first sentence), this is not the main conclusion of the argument. The author follows this observation with a counterpoint (“But…”), which she supports with the premise in the last sentence of the stimulus.

Answer choice (E): This answer choice is provably true but incorrect, because it functions as a premise for the economist’s conclusion.

Get the most out of your LSAT Prep Plus subscription.

Analyze and track your performance with our Testing and Analytics Package.